Download as pdf or txt
Download as pdf or txt
You are on page 1of 5

DEPARTMENT OF ELECTRICAL ENGINEERING

NATIONAL INSTITUTE OF TECHNOLOGY, ROURKELA


Assignments-4, Session: 2023 - 2024 (Spring)
B. Tech 6th Semester
Communication Systems (Code: EE3402)

Answer all questions

1. Sketch ϕF M (t) and ϕP M (t) for the modulating signal m(t) shown in Fig., given ωc = 2π ×107, kf = 104π,
and kp = 25π .

2. A baseband signal m(t) is the periodic sawtooth signal shown in Fig.

(a) Sketch ϕF M (t) and ϕP M (t) for this signal m(t) if ωc = 2π × 106 , kf = 2000π , and kp = π/2.
(b) Show that the PM signal is a signal with constant frequency but periodic phase changes.

Explain why it is necessary to use kp < π in this case for reception purposes. [Note that the PM signal
has a constant frequency but has phase discontinuities corresponding to the discontinuities of m(t).]

3. A periodic message signal m(t) as shown in Figure is transmitted as an angle-modulated signal.

The modulation system has ωc = 2π × 103 rad/s. Let the signal bandwidth of m(t) be approximated by
5 divided by its own period (i.e., its fifth harmonic frequency).

(a) To generate an FM signal with kf = 20π , sketch the frequency-modulated signal ϕF M (t) in the
time domain.
(b) If a PM signal is generated for m(t) with kp = π/2, sketch the phase-modulated signal ϕP M (t) in
the time domain.

4. Over an interval |t| ≤ 1, an angle-modulated signal is given by

ϕEM (t) = 10cos(13, 000πt + 0.3π)

It is known that the carrier frequency ωc = 12, 000π .

(a) Assuming the modulated signal is a PM signal with kp = 1000, determine m(t) over the interval
|t| ≤ 1.
(b) Assuming the modulated signal is an FM signal with kf = 1000, determine m(t) over the interval
|t| ≤ 1.

5. A periodic message signal m(t) as shown in Figure is to be transmitted by using angle modulation. Its
bandwidth is approximated by 200 Hz. The modulation system has ωc = 4π × 103 rad/s.

(a) If an FM signal with kf = 500π is to be generated, sketch in the FM signal in the time domain.
(b) If a PM signal with kp = 0.25π is to be generated, sketch the PM signal waveform in the time
domain.

6. For a message signal √


m(t) = 3 cos 1000t − 5 cos 1200 2πt

(a) Write expressions (do not sketch) for φPM (t) and φFM (t) when A = 10, ωc = 106 , kf = 1000π, and
kp = 1. For determining φFM (t), use the indefinite integral of m(t); that is, take the value of the
integral at t = −∞ to be 0 .
(b) Estimate the bandwidths of φFM (t) and φPM (t).

7. An angle-modulated signal with carrier frequency ωc = 2π × 106 is

φEM (t) = 10 cos (ωc t + 0.1 sin 2000πt)

(a) Find the power of the modulated signal.


(b) Find the frequency deviation ∆f .
(c) Find the phase deviation ∆ϕ.
(d) Estimate the bandwidth of φEM (t).

8. An angle-modulated signal with carrier frequency ωc = 2π × 106 is

φEM (t) = 5 cos (ωc t + 20 cos 1000πt + 10 sin 4000t)

(a) Find the power of the modulated signal.


(b) Find the frequency deviation ∆f .
(c) Find the phase deviation ∆ϕ.
(d) Estimate the bandwidth of φEM (t).

9. Given m(t) = sin 2000πt, kf = 5, 000π, and kp = 10,

(a) Estimate the bandwidths of φFM (t) and φPM (t).


(b) Repeat part (a) if the message signal amplitude is doubled.
(c) Repeat part (a) if the message signal frequency is doubled.
(d) Comment on the sensitivity of FM and PM bandwidths to the spectrum of m(t).
2
10. Given m(t) = e−100t , fc = 104 Hz, kf = 500π, and kp = 1.2π.

(a) Find ∆f , the frequency deviation for FM and PM.


(b) Estimate the bandwidths of the FM and PM waves. Hint: Find M (f ) first and find its 3 dB
bandwidth.

11. (a) Show that when m(t) has no jump discontinuities, an FM demodulator followed by an integrator
(Fig. a) forms a PM demodulator. Explain why it is necessary for the FM demodulator to remove
any dc offset before the integrator.
(b) Show that a PM demodulator followed by a differentiator (Fig. b) serves as an FM demodulator
even if m(t) has jump discontinuities or even if the PM demodulator output has dc offset.

12. A periodic square wave m(t) (Fig. a) frequency-modulates a carrier of frequency fc = 10 kHz with
∆f = 1 kHz. The carrier amplitude is A. The resulting FM signal is demodulated, as shown in Fig. b.
Sketch the waveforms at points b, c, d, and e.
13. Let s(t) be an angle-modulated signal that arrives at a receiver,

s(t) = 2 cos[107πt + 2 sin(1000πt + 0.3π) − 3πcos(2000t)]

(a) Find the bandwidth of this FM signal.


(b) If s(t) is sent to an (ideal) envelope detector, find the detector output signal.
(c) If s(t) is first differentiated before the envelope detector, find the detector output signal.
(d) Explain which detector output can be processed to yield the message signal m(t) and find the
message signal m(t) if kf = 200π.

14. A transmitter transmits an AM signal with a carrier frequency of 1470kHz. When a superheterodyne
radio receiver (which has a poor selectivity in its RF-stage bandpass filter) is tuned to 1530kHz, the
signal is heard loud and clear. However, if this radio is cheap, its front-end bandpass filter is not very
selective. Thus, the same signal is also heard (not as strongly) when tuned to another carrier frequency
setting within the AM range of 535-1605 kHz. State, with reasons, at what frequency you will hear this
station. The IF is 455kHz.

15. Consider a superheterodyne FM receiver designed to receive the frequency band of 88 to 108MHz with IF
of 10.7MHz. What is the range of frequencies generated by the local oscillator for this receiver? Analyze
and explain whether it is possible for an FM receiver to receive both a desired FM station and an image
FM station when tuned to the desired frequency.

16. In shortwave AM radio, the IF is also 455kHz. A receiver is designed to receive shortwave broadcasting
of 25 -meter band between 11.6 and 12.1MHz.

(a) Determine the frequency range of the local oscillator for this receiver.
(b) Analyze and explain whether it is possible for this receiver to receive both a desired AM station and
an image station within the same 25 -meter band.

17. (a) Design (the block diagram of) an Armstrong indirect FM modulator to generate an FM carrier
with a carrier frequency of 98.1MHz and ∆f = 75kHz. A narrowband FM generator is available
at a carrier frequency of 100kHz and a frequency deviation ∆f = 10 Hz. The stockroom also has
an oscillator with an adjustable frequency in the range of 10 to 11MHz. There are also plenty of
frequency doublers, triplers, and quintuplers.
(b) Determine the tunable range of the carrier frequency in the design of part (a).

18. Design (the block diagram of) an Armstrong indirect FM modulator to generate an FM carrier with
a carrier frequency of 96MHz and ∆f = 20kHz. A narrowband FM generator with fC = 200kHz and
adjustable ∆f in the range of 9 to 10 Hz is available. The stockroom also has an oscillator with adjustable
frequency in the range of 9 to 10MHz. There are bandpass filters with any center frequency, and only
frequency doublers are available.

19. Design an Armstrong indirect FM modulator in block diagram to generate an FM signal with carrier
96.3 MHz and ∆f = 20.48 kHz. A narrowband FM generator with fc = 150 kHz and ∆f = 10 Hz
is available. Only a limited number of frequency doublers are available as frequency multipliers. In
addition, an oscillator with adjustable frequency from 13 to 14 MHz is also available for mixing, along
with bandpass filters of any specification.

20. Use small-error analysis of PLL to show that a first-order loop [H(s) = 1] cannot track an incoming signal
whose instantaneous frequency varies linearly with time [θi (t) = kt2 ]. This signal can be tracked within
a constant phase if H(s) = (s + a)/s. It can be tracked with zero phase error if H(s) = (s2 + as + b)/s2 .
21. A second-order PLL is implemented with a nonideal loop filter
s+a
H(s) =
s+b

in which b > 0 is very small.

(a) Applying small-signal analysis, determine the transfer function between e(s) and i(s).
(b) Find the steady state PLL phase error for an incoming phase

θi (t) = (ω0 − ωc )t + ϕ0

You might also like